Orthogonal complement of a subspace

Click For Summary
To find the orthogonal complement of the subspace W spanned by the vectors w1, w2, and w3, the transpose of the matrix formed by these vectors is used to determine the null space. The row reduction of the transposed matrix yields the null space as span{[-4, 1, 0, 3], [-3, 0, 1, 0]}. The confusion arises regarding the expected dimension of W perp, which is calculated as 4 minus the dimension of W. The dimension of W is not necessarily three if the vectors are not linearly independent, as demonstrated by the relation 3*w1 + 4*w2 = w3. Thus, the assumption that the three vectors form a basis for W is incorrect due to their linear dependence.
bcjochim07
Messages
366
Reaction score
0

Homework Statement


Let W be the subspace spanned by the given column vectors. Find a basis for W perp.

w1= [2 -1 6 3] w2 = [-1 2 -3 -2] w3 = [2 5 6 1]
(these should actually be written as column vectors.




Homework Equations





The Attempt at a Solution



So, I put these vectors into a matrix and took its transpose since the orthogonal complement of the column space of a matrix equals the null space of the transpose.

I row reduced the transpose and got null(A transpose) = span{ [-4 1 0 3] , [-3 0 1 0]}
(Again, these should be written as column vectors)

This is the correct answer, but I thought that I should have gotten a null space with dimension one. The three vectors that span W "live" in R4 and the basis for W has dimension three. 4 - 3 =1, so shouldn't the dimension of W perp = 1?
 
Physics news on Phys.org
You're sure that w1, w2 and w3 actually span a 3 dimensional space, are you?
 
That would only be true if the three original vectors were linearly independent (i.e., if the space they span had dimension 3). Is this true?
 
Ok, I see. 3*w1 + 4*w2 = w3. I guess I just assumed that when it said a subspace W spanned by the vectors w1,w2,& w3, that those vectors would form a basis for W.
 
Question: A clock's minute hand has length 4 and its hour hand has length 3. What is the distance between the tips at the moment when it is increasing most rapidly?(Putnam Exam Question) Answer: Making assumption that both the hands moves at constant angular velocities, the answer is ## \sqrt{7} .## But don't you think this assumption is somewhat doubtful and wrong?

Similar threads

Replies
15
Views
2K
  • · Replies 4 ·
Replies
4
Views
2K
  • · Replies 15 ·
Replies
15
Views
3K
Replies
8
Views
2K
  • · Replies 4 ·
Replies
4
Views
2K
Replies
8
Views
2K
  • · Replies 1 ·
Replies
1
Views
2K
  • · Replies 1 ·
Replies
1
Views
1K
  • · Replies 14 ·
Replies
14
Views
2K
Replies
1
Views
1K